返回列表 发帖

请教一道逻辑题,谢谢

答案好象不对. 请各位帮忙看看.

The program to control the entry of illegal drugs into the country was a failure in 1987. If the program had been successful, the wholesale price of most illegal drugs would not have dropped substantially in 1987.
The argument in the passage depends on which of the following assumptions?
A. The supply of illegal drugs dropped substantially in 1987.
B. The price paid for most illegal drugs by the average consumer did not drop substantially in 1987.
C. Domestic production of illegal drugs increased at a higher rate than did the entry of such drugs into the country.
D. the wholesale price of a few illegal drugs increased substantially in 1987
E. A drop in demand for most illegal drugs in 1987 was not the sole cause of the drop in their wholesale price.


the answer is B. however, I think E is much better?
收藏 分享

The point is:
the programm attempts to control the ENTRY but DEMAND of illegal drugs in the country.
E points out a drop in demand, is an irrelevant choice.
Hope it helps.


[此贴子已经被作者于2002-9-2 8:55:39编辑过]

TOP

I think E is correct answer. would you check it again? B is irrelevant.

还请各位指教.

TOP

The answer is E

TOP

这是OG第80题,ets的答案是E

TOP

E is right

TOP

返回列表

站长推荐 关闭


美国top10 MBA VIP申请服务

自2003年开始提供 MBA 申请服务以来,保持着90% 以上的成功率,其中Top10 MBA服务成功率更是高达95%


查看